Thinking Critically and Solving Problems
About how much did the percent of working women with some college or an associate degree change
from 1996 to 2016?
Use the graph to answer the question.
Percent of women in the labor
force by educational attainment
100%
OOOO
A) 0%
B) 8%
C) 12%
D) 70%
80%
60%
Less than a high school diploma
High school graduates, no college
Some college or associate degree
Bachelor's degree and higher
40%
20%
0%
SUBMIT
1996
2006
2016
Source: U.S. Bureau of Labor Statistics
Question 16 of 21
31 AM

Thinking Critically And Solving ProblemsAbout How Much Did The Percent Of Working Women With Some College

Answers

Answer 1

Answer:maybe B?

Step-by-step explanation:


Related Questions

The function f is defined by the following rule. f(x) = 5x+1 Complete the function table.

Answers

Answer:

[tex]-5 \to -24[/tex]

[tex]-1 \to -4[/tex]

[tex]2 \to 11[/tex]

[tex]3 \to 16[/tex]

[tex]4 \to 21[/tex]

Step-by-step explanation:

Given

[tex]f(x) = 5x + 1[/tex]

Required

Complete the table (see attachment)

When x = -5

[tex]f(-5) = 5 * -5 + 1 = -24[/tex]

When x = -1

[tex]f(-1) = 5 * -1 + 1 = -4[/tex]

When x = 2

[tex]f(2) = 5 * 2 + 1 = 11[/tex]

When x = 3

[tex]f(3) = 5 * 3 + 1 = 16[/tex]

When x = 4

[tex]f(4) = 5 * 4 + 1 = 21[/tex]

So, the table is:

[tex]-5 \to -24[/tex]

[tex]-1 \to -4[/tex]

[tex]2 \to 11[/tex]

[tex]3 \to 16[/tex]

[tex]4 \to 21[/tex]

Joe bikes at the speed of 30 km/h from his home toward his work. If Joe's wife leaves home 5 mins later by car, how fast should she drive in order to overtake him in 10 minutes.

Answers

Answer:

Joe's wife must drive at a rate of 45km/hour.

Step-by-step explanation:

We are given that Joe leaves home and bikes at a speed of 30km/hour. Joe's wife leaves home five minutes later by car, and we want to determine her speed in order for her to catch up to Joe in 10 minutes.

Since Joe bikes at a speed of 30km/hour, he bikes at the equivalent rate of 0.5km/min.

Then after five minutes, when his wife leaves, Joe is 5(0.5) or 2.5 km from the house. He will still be traveling at a rate of 0.5km/min, so his distance from the house can be given by:

[tex]2.5+0.5t[/tex]

Where t represents the time in minutes after his wife left the house.

And since we want to catch up in 10 minutes, Joe's distance from the house 10 minutes after his wife left will be:

[tex]2.5+0.5(10)=7.5\text{ km}[/tex]

Let s represent the wife's speed in km/min. So, her speed times 10 minutes must total 7.5 km:

[tex]10s=7.5[/tex]

Solve for s:

[tex]\displaystye s=0.75\text{ km/min}[/tex]

Thus, Joe's wife must drive at a rate of 0.75km/min, or 45km/hour.

Last softball season, Pamela had 46 hits, a combination of singles (1 base), doubles (2 bases), and triples (3 bases). These 46 hits totaled 66 bases, and she had 4 times as many singles as doubles. How many doubles did she have?

Answers

Answer:

She had 8 doubles.

Step-by-step explanation:

This question is solved by a system of equations.

I am going to say that:

x is the number of singles.

y is the number of doubles

z is the number of triples.

46 hits

This means that [tex]x + y + z = 46[/tex]

46 hits totaled 66 bases

This means that:

[tex]x + 2y + 3z = 66[/tex]

4 times as many singles as doubles

This means that [tex]x = 4y[/tex]

So

[tex]x + 2y + 3z = 66[/tex]

[tex]4y + 2y + 3z = 66[/tex]

[tex]6y + 3z = 66[/tex]

And

[tex]x + y + z = 46[/tex]

[tex]4y + y + z = 46[/tex]

[tex]5y + z = 46 \rightarrow z = 46 - 5y[/tex]

Then

[tex]6y + 3z = 66[/tex]

[tex]6y + 3(46 - 5y) = 66[/tex]

[tex]6y + 138 - 15y = 66[/tex]

[tex]9y = 72[/tex]

[tex]y = \frac{72}{9}[/tex]

[tex]y = 8[/tex]

She had 8 doubles.

The diagram shows triangle ABC.
С
Work out the sizes of angles x, y and z.
40°
110°
х
Z
A
В

Answers

Answer:

x=70

y=30

z=20

Step-by-step explanation:

x=180-110 (angles on a straight line)

y=180-110-40 (angle sum of triangle)

z= 180-90-70 (angle sum of triangle)

Answer:

x=70°

y=30°

z=20°

Step-by-step explanation:

x=180°-110°(anlges on a straight line)

x=70°

y+110°+40°=180°(sum of angles of triangle)

y+150°=180°

y=180°-150°

y=30°

z+x+90°=180°(sum of angles of triangle)

z+70°+90°=180°

z+160°=180°

z=180°-160°

z=20°

The sum of the first ten terms of an arithmetic progression consisting of

positive integer terms is equal to the sum of the 20th, 21st and 22nd term.

If the first term is less than 20, find how many terms are required to give

a sum of 960.

Answers

Answer: [tex]n=13[/tex]

Step-by-step explanation:

Given

Sum of the first 10 terms is equal to sum of 20, 21, and 22 term

[tex]\Rightarrow \dfrac{10}{2}[2a+(10-1)d]=[a+19d]+[a+20d]+[a+21d]\\\\\Rightarrow 5[2a+9d]=3a+60d\\\Rightarrow 10a+45d=3a+60d\\\Rightarrow 7a=15d[/tex]

No of terms to give a sum of 960

[tex]\Rightarrow 960=\dfrac{n}{2}[2a+(n-1)d]\\\\\Rightarrow 1920=n[2a+(n-1)\cdot \dfrac{7}{15}a]\\\\\Rightarrow 28,800=n[30a+7a(n-1)]\\\\\Rightarrow a=\dfrac{28,800}{n[30+7n-7]}\\\\\Rightarrow a=\dfrac{28,800}{n[23+7n]}[/tex]

Value of first term is less than 20

[tex]\therefore \dfrac{28,800}{n[23+7n]}<20\\\\\Rightarrow 28,800<20n[23+7n]\\\Rightarrow 0<460n+140n^2-28,800\\\Rightarrow 140n^2+460n-28,800>0\\\\\Rightarrow n>12.79\\\\\text{For integer value }\\\Rightarrow n=13[/tex]

Answer:

15

Step-by-step explanation:

In the previous answer halfway through they used the equation: 960 = (n÷2)×(2a+(n-1)×(7a÷15))

Using this equation we can substitute an number to replace n, the higher the number is the smaller a would be.

When we substitute 15 into a, then it leaves us with the answer to be a = 15 which is a positive integer and also is smaller than 20, this then let’s us know that 15 is how many terms can be summed up to make 960.

To double check this answer you can find that d = 7 by changing the a into 15 in the formula 7a/15 (found in the previous answer.

Then in the expression: (n÷2)×(2a+(n-1)×d)

substitute:

n = 14 (must be an even number for the equation to work)

a = 15

d = 7

This will give you an answer of 847, but this is only 14 terms as we changed n into 14. To add the final term you need to complete the following equation: 847+(a+(n-1)×d)

substituting:

n = 15

a = 15

d = 7

This will give you the answer of 960, again proving that it takes 15 terms to sum together to make the number 960.

I hope this has helped you.

P.S. Everything in the previous solution was right apart from the start of the last section and the answer

Find the gradient of the tangent line to the curve y=-x² + 3x at the point (2, 2).​

Answers

Answer:

Y' = - 1

Step-by-step explanation:

Y' = - 2x +3

So y' (2,2) =-2*2 +3= - 1

A sample of 25 one-year-old girls had a mean weight of 24.1 pounds with a standard deviation of pounds. Assume that the population of weights is normally distributed. A pediatrician claims that the standard deviation of the weights of one-year-old girls is less than pounds. Do the data provide convincing evidence that the pediatrician's claim is true

Answers

Answer:

Paedtricians claim isn't true.

Step-by-step explanation:

The hypothesis :

H0 : σ = 7

H0 : σ > 7

The test statistic ; χ² :

χ² = [(n - 1) * s²] ÷ σ²

n = 25 ; s = 4.3, σ = 7

χ² = [(25 - 1) * 4.3²] ÷ 7²

χ² = [(24 * 4.3²] ÷ 49

χ² = 443.76 / 49

χ² = 9.056

At α = 0.01 ; critical value = 42.980

Since critical value > test statistic, we fail to reject the null, H0.

arrange the following in descending order - 5, 0, -15, 2.5, 2.05​

Answers

Answer:

2.5, 2.05, 0, -5, -15

Step-by-step explanation:

for negative numbers the bigger is worth less

Write down 4 pairs of integers a and b such that a divided by b is -5

Answers

1. 20 and -4
2. 15 and -3
3. 10 and -2
4. 5 and -1

Find the range of the data.
Scores: 81, 79, 80, 88, 72, 96, 86, 73, 79, 88

Answers

Answer:

24

Step-by-step explanation:

To find the range, you must subtract the lowest value from the highest value in the data set. If you organize the set from least to greatest, 72 is the lowest, and 96 is the highest.

So, 96 - 72 = 24, which is the range.

Suppose that you are thinking about buying a car and have narrowed down your choices to two options.
The new-car option: The new car costs $25,000 and can be financed with a four-year loan at 6.12%.
The used-car option: A three-year old model of the same car costs $17,000 and can be financed with a three-year loan at 7.72%.
=||)
[1-(2-4) 11
What is the difference in monthly payments between financing the new car and financing the used car? Use PMT
The difference in monthly payments between financing the new car and financing the used car is $
(Round to the nearest cent as needed.)

Answers

Answer:

sjsjsuduhr r ki snsbtsuwi 3 38yv4r djvs

Which expression is equivalent to 3 square root of x^5*y


Answers

Answer:

√3 x^5y

First, let's do √3

√3=1.7

1.7 • x^5 • y

if you want

1.7 • X^4• x• y.

There are tons of equivalent's!

A school contains 140 boys and 160 girls. what is the ratio of boys to girls?
I need full working out please

Answers

Answer:

7 : 8

Step-by-step explanation:

that is the procedure above

What does y equal in the solution of the system of equations below? 5y-3x-4z=22 2z-2x=-6 2z+3x=-6

Answers

9514 1404 393

Answer:

  y = 2

Step-by-step explanation:

Subtracting the second equation from the third gives ...

  (2z +3x) -(2z -2x) = (-6) -(-6)

  5x = 0

  x = 0

Using this in the third equation, we have ...

  2z +0 = -6

  z = -3

And substituting these values into the first equation, we have ...

  5y -3(0) -4(-3) = 22

  5y = 10 . . . . . subtract 12

  y = 2

__

The solution to the system is (x, y, z) = (0, 2, -3).

which of the following function shows the absolute value parent function FX=lxl shifted up

Answers

Answer:

The answer is C.

as for C . the value of f(x) increases by 7 and so the graph goes up by units 7.

OR

g(x) = |x| + 7

we know that |x| is f(x), so :-

g(x) = f(x) + 7

and since f(x) is plot on y- axis the graph climbs the y axis by 7 units

*The graph shifts right or left for the other functions*

Tessa conducts an experiment and obtains results that are statistically significant. What is meant by "statistically significant"? O It means that Tessa used a large sample size. O It means that the results that Tessa obtained are too unusual to be explained by chance alone. It means that Tessa's experiment was not biased. O It means that the individuals in the experiment were randomly assigned to the treatment groups. earch 0 so DOLL​

Answers

9514 1404 393

Answer:

  (b)  It means that the results that Tessa obtained are too unusual to be explained by chance alone

Step-by-step explanation:

The test for statistical significance is a comparison of the result to the probability that it occurred by chance. The probability of a chance occurrence is usually set at 1% or 5%. The lower the percentage, the less likely a chance occurrence is, and the more difficult showing statistical significance becomes.

SOMEONE HELP ASAP PLES NO EXPLANATOIN NEEDED PLS LEAVE UR ANSWER AS TEXT (SOME TIMES I CAN'T SEE IMAGES) THANK YOU SO MUCH!!!

Answers

Answer:

i cant see the image

Step-by-step explanation:

Which simplified fraction is equal to 0.53? Need answers now plz

Answers

Answer:

8/15

Step-by-step explanation:

Answer:

8/15

Step-by-step explanation:

when you divide 8/15 its 0.53

Please HELP!

How many pairs (A, B) are there where A and B are subsets of {1, 2, 3, 4, 5, 6, 7, 8} and A ∩ B has exactly two elements?

Answers

Answer:

There are 256 pairs in all.

Find the solution of the differential equation that satisfies the given initial condition. (dP)/(dt)

Answers

Answer:

[tex]P = (\frac{1}{3}t^\frac{3}{2} + \sqrt 2 - \frac{1}{3})^2[/tex]

Step-by-step explanation:

Given

[tex]\frac{dP}{dt} = \sqrt{Pt[/tex]

[tex]P(1) = 2[/tex]

Required

The solution

We have:

[tex]\frac{dP}{dt} = \sqrt{Pt[/tex]

[tex]\frac{dP}{dt} = (Pt)^\frac{1}{2}[/tex]

Split

[tex]\frac{dP}{dt} = P^\frac{1}{2} * t^\frac{1}{2}[/tex]

Divide both sides by [tex]P^\frac{1}{2}[/tex]

[tex]\frac{dP}{ P^\frac{1}{2}*dt} = t^\frac{1}{2}[/tex]

Multiply both sides by dt

[tex]\frac{dP}{ P^\frac{1}{2}} = t^\frac{1}{2} \cdot dt[/tex]

Integrate

[tex]\int \frac{dP}{ P^\frac{1}{2}} = \int t^\frac{1}{2} \cdot dt[/tex]

Rewrite as:

[tex]\int dP \cdot P^\frac{-1}{2} = \int t^\frac{1}{2} \cdot dt[/tex]

Integrate the left hand side

[tex]\frac{P^{\frac{-1}{2}+1}}{\frac{-1}{2}+1} = \int t^\frac{1}{2} \cdot dt[/tex]

[tex]\frac{P^{\frac{-1}{2}+1}}{\frac{1}{2}} = \int t^\frac{1}{2} \cdot dt[/tex]

[tex]2P^{\frac{1}{2}} = \int t^\frac{1}{2} \cdot dt[/tex]

Integrate the right hand side

[tex]2P^{\frac{1}{2}} = \frac{t^{\frac{1}{2} +1 }}{\frac{1}{2} +1 } + c[/tex]

[tex]2P^{\frac{1}{2}} = \frac{t^{\frac{3}{2}}}{\frac{3}{2} } + c[/tex]

[tex]2P^{\frac{1}{2}} = \frac{2}{3}t^\frac{3}{2} + c[/tex] ---- (1)

To solve for c, we first make c the subject

[tex]c = 2P^{\frac{1}{2}} - \frac{2}{3}t^\frac{3}{2}[/tex]

[tex]P(1) = 2[/tex] means

[tex]t = 1; P =2[/tex]

So:

[tex]c = 2*2^{\frac{1}{2}} - \frac{2}{3}*1^\frac{3}{2}[/tex]

[tex]c = 2*2^{\frac{1}{2}} - \frac{2}{3}*1[/tex]

[tex]c = 2\sqrt 2 - \frac{2}{3}[/tex]

So, we have:

[tex]2P^{\frac{1}{2}} = \frac{2}{3}t^\frac{3}{2} + c[/tex]

[tex]2P^{\frac{1}{2}} = \frac{2}{3}t^\frac{3}{2} + 2\sqrt 2 - \frac{2}{3}[/tex]

Divide through by 2

[tex]P^{\frac{1}{2}} = \frac{1}{3}t^\frac{3}{2} + \sqrt 2 - \frac{1}{3}[/tex]

Square both sides

[tex]P = (\frac{1}{3}t^\frac{3}{2} + \sqrt 2 - \frac{1}{3})^2[/tex]

A 27​% solution ​( 27mg per 100 mL of​ solution) is given intravenously. Suppose a total of 1,36 L of the solution is given over a 10 ​-hour period. Complete parts​ (a) through​ (c) below.
a. What is the flow rate in units of​ mL/hr?
nothing ​mL/hr ​(Type an integer or decimal rounded to the nearest thousandth as​ needed.)
What is the flow rate in per​ hour?
nothing ​mg/hr ​(Type an integer or decimal rounded to the nearest thousandth as​ needed.)
b. If each mL contains 13 drops​ (the drop factor is expressed as ​gtt/mL), what is the flow rate in units of​ 13gtt/hr?
nothing ​gtt/hr ​(Type an integer or decimal rounded to the nearest thousandth as​ needed.)
c. During the 10 ​-hour ​period, how much is​ delivered?
nothing mg ​(Type an integer or decimal rounded to the nearest thousandth as​ needed.)

Answers

Answer:

Step-by-step explanation:

a.

(1.36 L)/(10 hr) = (0.136 L)/(hr)

Flow rate = (0.136 L)/(hr) × (1000 mL)/L = (136 mL)/(hr)

136 mL × (27 mg)/(100 mL) = 36.72 mg

Delivery rate = (36.72 mg)/(hr)

b.

(136 mL)/(hr) × (13 gtt)/(mL) = (1868 gtt)/(hr)

c.

10 hr × (36.72 mg)/)hr) = 367.2 mg

Private nonprofit four-year colleges charge, on average, $26,208 per year in tuition and fees. The standard deviation is $7,040. Assume the distribution is normal. Let X be the cost for a randomly selected college. Round all answers to 4 decimal places where possible.

a. What is the distribution of X? X ~ N(
26208
Correct,
7040
Correct)

b. Find the probability that a randomly selected Private nonprofit four-year college will cost less than 22,924 per year.


c. Find the 60th percentile for this distribution. $
(Round to the nearest dollar.)

Answers

Answer:

#########

Step-by-step explanation:

If a line is equation 5 x + 6 Y is equal to 2 k together with the coronavirus access from a triangle of area 135 square unit.find the value of k.

Answers

Answer:

hdhdkdbddkdgsjshzcssjsn

How would 0.42 be shown as a percent?
A. 0.42%
B. 4%
C. 4.2%
D. 42%

Answers

Answer:

42%

Step-by-step explanation:

to find percentages, you move the decimal point twice to the right

Let f(x)
2x + 8, g(x) = x2 + 2x – 8, and h(x) = 3x – 6.
Perform the indicated operation. (Simplify as far as possible.)
(h · f)(3) =

Answers

Answer:

36

Step-by-step explanation:

(h · f)(x) = h(f(x))

h(f(x)) = h(2x+8)

h(f(x))= 3(2x+8) - 6

h(f(x)) = 6x + 24 - 6

h(f(x))= 6x + 18

If x = 3

h(f(x))= 6(3) + 18

h(f(x))= 18 + 18

h(f(x))= 36

Hence (h · f)(3) = 36

HELP
-5(2m-3)-4<81
I need the steps also well

Answers

Answer:

m>-7

Step-by-step explanation:

expand

-10m+15-4<81

-10m+11<81

collect like terms

-10m<81-11

-10m<70

m>-7

If 19,200 cm2 of material is available to make a box with a square base and an open top, find the largest possible volume of the box.

Answers

Step-by-step explanation:

√19200cm²

=138.56cm

then the highest possible volume

=(138.56)³

=2660195.926cm³

The largest possible volume of the box is; V = 25600 cm³

Let us denote the following of the square box;

Length = x

Width = y

height = h

Formula for volume of a box is;

V = length * width * height

Thus; V = xyh

but we are dealing with a square box and as such, the base sides are all equal and so; x = y. Thus;

V = x²h

The box has an open top and as such, the surface are of the box is;

S = x² + 4xh

We are given S = 19200 cm². Thus;

19200 = x² + 4xh

h = (19200 - x²)/4x

Put (19200 - x²)/4x for h in volume equation to get;

V = x²(19200 - x²)/4x

V = 4800x - 0.25x³

To get largest possible volume, it will be dimensions when dV/dx = 0. Thus;

dV/dx = 4800 - 0.75x²

At dV/dx = 0, we have;

4800 - 0.75x² = 0

0.75x² = 4800

x² = 4800/0.75

x² = 6400

x = √6400

x = 80 cm

From h = (19200 - x²)/4x;

h = (19200 - 80²)/(4 × 80)

h = (19200 - 6400)/3200

h = 4 cm

Largest possible volume = 80² × 4

Largest possible volume = 25600 cm³

Read more at; https://brainly.com/question/19053087


It takes Ricky, traveling at 24 mph, 45 minutes longer to go a certain distance than it takes Maria traveling at 51 mph, Find the distance traveled.

Answers

Answer:

85 mi

Step-by-step explanation:

Let d = the distance in miles traveled

Let M = the time in hours for Maria to travel d miles

[tex]m+\frac{3}{4} =[/tex] time in hours for Ricky to travel d miles

(Note that [tex]\frac{3}{4}[/tex] hrs = 45 min)

----------------------

Maria's equation:

d = 51m

Ricky's equation:

d = 24 · [tex](m+\frac{3}{4} )[/tex]

----------------------

Substitution:

51m = 24 · [tex](m+\frac{3}{4} )[/tex]

51m = 24m + 45

6m = 10

m = [tex]\frac{5}{3}[/tex]

----------------------

d = 51m

d = 51 · [tex](\frac{5}{3})[/tex]

d = 85

----------------------

The distance traveled is 85 mi

If it takes Ricky, traveling at 24 mph, 45 minutes longer to go a certain distance than it takes Maria traveling at 51 mph, the distance traveled is 85 miles

Speed and distances

Speed is the ratio of distance traveled to time taken. Mathematically:

Distance = Speed/Time

According to the given question:

Let d be the distance in miles traveledLet M be the time in hours for Maria to travel d milesLet the required time in hours for Ricky to travel be d miles

Set up the Maria equation:

d = 51m

Set up Ricky's equation:

d = 24 · (m+3/4)

Substitute

51m = 24 · (m+3/4)

51m = 24m + 45

6m = 10

m = 5/3

Determine the required distance

d = 51m

d = 51 · 5/3

d = 85

Hence the distance traveled is 85 mile

Learn more on distance and speed here: https://brainly.com/question/26046491

Meghan sells advertisements for a radio station. Each 30 second ad costs $20 per play, and each 60 second ad
costs $35 per play. Meghan sold 12 ads for $315. She wrote the system below letting x represent the number of 30
second ads and y represent the number of 60 second ads.
X+ y = 12
20x+35y = 315
What is the solution to the system of equations?

Need answers ASAP!!!!

Answers

Answer:

usai964s46s694s4o6s64694s946649s469 opps

Answer:

[tex](x,y)=(7,5)[/tex]

Step-by-step explanation:

Megan's equation will be:

[tex]20x+35y=315[/tex]

[tex]x+y=12[/tex]

Substitute [tex]x=12-y[/tex] in the first equation:

[tex]20(12-y)+35y=315[/tex]

[tex]15y=75[/tex]

[tex]y=75/15[/tex]

[tex]y=5[/tex]

Find x:

[tex]x=12-5[/tex]

[tex]x=7[/tex]

Where x and y represent 30-second and 60-second ads sold, we find that Meghan's sales were:

[tex](x,y)=(7,5)[/tex]

hope this helps....


Drag the tiles to the correct boxes to complete the pairs.
Match each division of rational expressions with its quotient.


Answers

Answer:

Step-by-step explanation:

Um where is the diagrahm

Other Questions
whch of the following can be usefiul when finding the vaue of a variable CWhich of the following did Federalists oppose?O acceptance of the US ConstitutionO the establishment of slavery in the United StatesO the inclusion of a bill of rights in the US ConstitutionO a strong national government with many powers (9-3)+188 show your work what are MA and VR of a lever? Alex likes to work alone. He has created a schedule, which he follows dutifully and allows himtime to write short stories. Alex's learning style is more likely:A environmentalB spatialC intrapersonalD kinesthetic Sam works at a shoe store. He earns $300 every week plus $15 for every pair of shoes that he sells. How many pairs of shoes would he need to sell to make $500 in a week? When a rigid body rotates about a fixed axis, all the points in the body have the same Group of answer choices linear displacement. angular acceleration. centripetal acceleration. tangential speed. tangential acceleration. Match the graph with the correct equation. A. Y-1 = -1/4(x+5) B. Y+1= -1/4(x+5) C. Y-1= -4(x+5) D. Y-1 =-1/4 (x-5) The marked price of a bicycle is Rs 2000. If the shopkeeper allows some discount and a customerbought it for Rs 1921 including 13% VAT, how much amount was given as the discount? He is well qualified, . he is still unemployed. (0.5 Points) 1 despite 2 although 3 however4 inspite of NEED HELP The average amount of money spent for lunch per person in the college cafeteria is $6.75 and the standard deviation is $2.28. Suppose that 18 randomly selected lunch patrons are observed. Assume the distribution of money spent is normal, and round all answers to 4 decimal places where possible.C. For a single randomly selected lunch patron, find the probability that this patron's lunch cost is between $7.0039 and $7.8026. D. For the group of 18 patrons, find the probability that the average lunch cost is between $7.0039 and $7.8026. Significant legacies of the civil war included all of the following except? A. The end of official slavery in the U.S B. Much of the American south destroyed C. A substantial reduction in government power because of the war D. Over 600,000 American combat deaths (more than every other American conflict combined) E. Triumph of the Northern system of politics, ideal, and economic Someone please help me with this algebra problem Hey Help me out please, thanks! Which of the following is included in an informed opinion? Sendo assim, podemos afirmar quepopulao do Brasil Assume the equation has a solution for z. -cz+ 6z = tz + 83z=? Life Expectancies In a study of the life expectancy of people in a certain geographic region, the mean age at death was years and the standard deviation was years. If a sample of people from this region is selected, find the probability that the mean life expectancy will be less than years. Round intermediate -value calculations to decimal places and round the final answer to at least decimal places. In Business ABC, the number of clients (buyers) increased after an advertisement campaign. The manager concludes that the number of clients increased because of the advertisement campaign.Discuss in detail why the managers conclusion can be wrong?How can we actually find out whether the advertisement was successful? Discuss in detail. Which is appropriate for the POL?